LSAT and Law School Admissions Forum

Get expert LSAT preparation and law school admissions advice from PowerScore Test Preparation.

 Administrator
PowerScore Staff
  • PowerScore Staff
  • Posts: 8926
  • Joined: Feb 02, 2011
|
#66060
Please post your questions below!
 lawadvocate!
  • Posts: 5
  • Joined: Jul 15, 2019
|
#66543
Hello!

I was just kind of lost when I looked at this question during the exam so I skipped it.

Stimulus breakdown:
Conclusion: The company must give Vernon his job back.
Why?
Evidence: Vernon's behavior was unprofessional enough that the company was justified in firing him. But other high-ranking employees who were involved in the incident & were just as unprofessional haven't been fired.

No idea how to tackle this one. My prephrase is that well what if the employees who weren't fired were not fired for another reason? Sure they behaved just as unprofessionally, but maybe they also contribute greatly to the company? Even more than Vernon. So the same treatment shouldn't apply just because they weren't fired? Maybe they do more/hold more weight in the company than Vernon?

Any explanation would be helpful. Thank you!
 Adam Tyson
PowerScore Staff
  • PowerScore Staff
  • Posts: 5164
  • Joined: Apr 14, 2011
|
#66583
Your analysis there is missing one crucial bit of information, lawadvocate!, and that is "for the sake of consistency" in the conclusion. The author doesn't just think Vernon should get his job back - he thinks that "consistency" requires that. But that fails to consider that there is another way to achieve consistency, which is to fire the other people who have behaved as Vernon did. That is a viable alternative to the one the author proposed that also achieves that consistency.

If Vernon's behavior was unprofessional enough to justify firing him, and others behaved the same way, that suggests pretty strongly that the behavior should override any other considerations. It doesn't matter how good an employee he is, or they are, or how much any of them contribute - his behavior was sufficient, so theirs should be too. Sure, those other folks might be valuable in some way that Vernon was not, but he also might have been valuable in some way that they were not. We just can't assume anything either way.

What we can do is look at the alternatives, and the author left out a fairly obvious one. Don't bring back the guy who was justly fired - fire the guys who were just as bad as that guy was!

Beware the False Dilemma here, where the author presumes that one particular choice must be made without giving proper attention to other potential choices. "It's not getting hotter, so it must be getting colder" leaves out the possibility of the temperature holding steady. "I won't drink Pepsi, so I have to have Coke" leaves out a whole world of other possible options, including other colas, sodas that are not colas, and all the non-soda options out there, as well as not having anything to drink. Both of these are variations on the False Dilemma flaw that we sometimes see on the LSAT.
User avatar
 Canadianlaw1100
  • Posts: 2
  • Joined: Dec 20, 2020
|
#82715
Hello!

I understand why E is right, but I cant seem to understand why D (what I originally picked) is wrong..
User avatar
 KelseyWoods
PowerScore Staff
  • PowerScore Staff
  • Posts: 1079
  • Joined: Jun 26, 2013
|
#82798
Hi Canadianlaw1100!

Answer choice (D) reads: "treating behavior that can sometimes result in a certain consequence as behavior that always results in that consequence." This describes an overgeneralization flaw and it doesn't match what is happening in the stimulus. To match this answer choice, the argument in the stimulus would have to say something along the lines of "unprofessional behavior sometimes results in firing" and then conclude "therefore, unprofessional behavior always results in firing." But this is not an accurate description of the premises and the conclusion that we have in the stimulus. And, in fact, the conclusion of the stimulus is not about what consequence will result, it's about something that should happen (giving Vernon back his job).

For Flaw questions, first make sure that the answer choice accurately describes the reasoning that is taking place in the stimulus argument (how the premises are supporting the conclusion) and then make sure that it describes why that reasoning is flawed. Map the abstract statements in the answer choice to the specific statements in the stimulus to make sure they match.

Hope this helps!

Best,
Kelsey
User avatar
 ec623ec623
  • Posts: 6
  • Joined: Mar 21, 2023
|
#101969
I originally picked E, but then switched to B because I didn't see "another equally supported response" in the stimulus. Reading all of the explanations above I can see that my thought process was on the right track for my initial answer. I think I may have tripped up on the "equally supported" part because that made me think that it would need support from the stimulus.

For B, I think my thought process was thinking that unprofessional behavior is sufficient to fire someone, but not necessary. I'm thinking maybe it was wrong because it doesn't support the "consistency" part in conclusion but I'm not sure. Could you please explain more about how B is wrong?
 Luke Haqq
PowerScore Staff
  • PowerScore Staff
  • Posts: 763
  • Joined: Apr 26, 2012
|
#101993
Hi ec623ec623!

Regarding (B), there doesn't seem to be manifest confusion over sufficient versus necessary conditions in the stimulus.

Rather, there's a question of why other employees were treated differently in comparison with Vernon. More to the point, the conclusion is about what consistency requires, given this differential treatment. You're right to conclude that answer choice (B) doesn't quite get to how consistency functions in the conclusion of the argument. The conclusion is that, "for the sake of consistency, the company must give Vernon his job back." While giving him his job back is one way to ensure consistency, another way is to fire all the other mentioned employees who acted similarly. The argument doesn't consider this possibility, however--and in this respect, it fails to consider "another equally supported response," as reflected in (E).
User avatar
 ihenson
  • Posts: 8
  • Joined: Jul 02, 2023
|
#102690
KelseyWoods wrote: Mon Dec 28, 2020 3:33 pm Hi Canadianlaw1100!

Answer choice (D) reads: "treating behavior that can sometimes result in a certain consequence as behavior that always results in that consequence." This describes an overgeneralization flaw and it doesn't match what is happening in the stimulus. To match this answer choice, the argument in the stimulus would have to say something along the lines of "unprofessional behavior sometimes results in firing" and then conclude "therefore, unprofessional behavior always results in firing." But this is not an accurate description of the premises and the conclusion that we have in the stimulus. And, in fact, the conclusion of the stimulus is not about what consequence will result, it's about something that should happen (giving Vernon back his job).

For Flaw questions, first make sure that the answer choice accurately describes the reasoning that is taking place in the stimulus argument (how the premises are supporting the conclusion) and then make sure that it describes why that reasoning is flawed. Map the abstract statements in the answer choice to the specific statements in the stimulus to make sure they match.

Hope this helps!

Best,
Kelsey
Hi Kelsey,

I'm a confused by this explanation. Isn't it what is happening in the stimulus? The employee is taking something that sometimes happens (getting fired after doing something unprofessional) and trying to apply it to every situation? There by making it something that "always" has to happen?

Would D be a better answer if it said "treating behavior that can sometimes results in a certain consequence as behavior that should always result in that consequence?"
 Adam Tyson
PowerScore Staff
  • PowerScore Staff
  • Posts: 5164
  • Joined: Apr 14, 2011
|
#102768
That certainly would make D a better answer, ihenson! Although I'm still not sure it would be as good as E. After all, the people who were not fired did NOT have a consequence; they were free from consequences. So really, the author isn't saying that behavior that can sometimes result in a certain consequence should always results in that consequence; they are saying that behavior that sometimes has no consequences should never have consequences.

And it's not entirely clear that either of those would really be a flaw, since the author is focused on being consistent. That implies an all-or-nothing approach would be valid. The real problem is that the author presumes that "nothing" is the right solution, which fails to consider that "all" is an equally valid choice.
 joliekwok8@gmail.com
  • Posts: 14
  • Joined: Jun 21, 2023
|
#102822
Hi,

I just wanted to break down what answer C which says "offering as its primary evidence a premise that is equivalent to the argument's conclusion" is supposed to mean? In my mind, this described what the argument was doing since the employee's conclusion is that consistency is the main reason why Vernon should get his job back and in the past (which is also the premise), other employees have not been fired for doing the same thing.

I'm wondering why this isn't considered a flaw as well, since (at least in my mind) the employee is just using past experiences to justify the employee's conclusion? I thought this was similar to the "the sun rises every day so it must rise tomorrow" kind of argument? Though even now as I type this out, there are some key differences between the two...

But for now i'm just wondering if my interpretation of what answer C is describing is correct.

Thank you!

Get the most out of your LSAT Prep Plus subscription.

Analyze and track your performance with our Testing and Analytics Package.